LSAT and Law School Admissions Forum

Get expert LSAT preparation and law school admissions advice from PowerScore Test Preparation.

 Administrator
PowerScore Staff
  • PowerScore Staff
  • Posts: 8929
  • Joined: Feb 02, 2011
|
#40619
Complete Question Explanation
(The complete setup for this game can be found here: lsat/viewtopic.php?t=15461)

The correct answer choice is (A)

This question also puts the 3-2-1 distribution into effect, with exactly one dish stored on the middle shelf:

PT69_Game_#2_#11_diagram 1.png
The first inference that can be drawn is that dish 2 must be stored on the top shelf. This occurs due to the combination of the second rule and the inference that the middle shelf must always contain dish 5 or dish 6. For example, if dish 6 is stored on the bottom shelf, then dish 5 is the only dish stored on the middle shelf, forcing dish 2 to be stored on the top shelf. If dish 6 is stored on the middle shelf, then dish 2 must be stored on the top shelf. Thus, dish 2 must always be stored on the top shelf in this question. This inference alone eliminates answer choices (B), (D), and (E), each of which do not contain dish 2.

In addition, from the fourth rule, dishes 1 and 4 cannot be stored on the same shelf, and since the only two shelves with open space are the top and bottom shelves, dishes 1 and 4 must occupy a space on each shelf.

The two inferences just discussed can be added to the diagram:

PT69_Game_#2_#11_diagram 2.png
Because each of the answer choices lists only two dishes, the correct answer must be either dishes 2 and 1, or dishes 2 and 4. Accordingly, answer choice (A) is correct.
You do not have the required permissions to view the files attached to this post.
 MichaelJAG
  • Posts: 12
  • Joined: Aug 21, 2015
|
#20164
Dear PowerScore,

This problem is quite confusing. I was able to successfully workout all of the answer choices except for (E).

T/B: 1/4 2 5/3
M: 6/5
B/T: 4/1 3/5/6

From this main setup:

(A) T: 1 2 3
M: 6
B: 5 4

(B) T: 1 5 2
M: 6
B: 4 3

(C) T: 2 3 1 (This is basically the solution to number 8, since the top shelf is constant in both
M: 6
B: 5 4

(D) T: 3 4 2
M: 6
B: 5 1

(E) T: 3 5 2
M: 6
B: 1 4 (This violates the fourth rule)

PowerScore explanations said that, since either 5 or 6 must go in the middle, then 2 must go on the top shelf, which makes perfect sense. However, they then say that that inference eliminates answer choices B, D and E, because they do not contain 2. However, the answer choices only require two of the numbers on the top shelf, yet nothing in the question makes me assume that the top shelf only has two numbers, so how do we know that the top shelf does not hold 3 dishes? Even still, if 2 must be on the top shelf, then how does that mean that 3 cannot go there (answer choice D)?: There are not even rules to restrict its placement. I must have missed something huge, but I read the PowerScore explanation multiple times, and I still come up with the same scenario. Thank-you.

Regards,

Michael
 Ricky_Hutchens
PowerScore Staff
  • PowerScore Staff
  • Posts: 59
  • Joined: Oct 12, 2015
|
#20196
Hi Michael,

I applaud your perseverance. The good news is that this is the type of mistake that once it has been pointed out to you, is easily correctable.

The reason the explanation says that you can eliminate answer choices B, D, E is because the question asks for "the list stored on the top shelf." That means the top shelf will be limited to whatever is contained in the answer choice. If it wasn't, then the answer choice would not be the list of items on the top shelf, but only a portion of that list.

It's important to note that the questions does not require that there be only 2 dishes on the top shelf, which is why this is a could be true questions instead of must be true. But the language of the question does limit what is on the top shelf to whatever is in the answer choice.

So since dish 2 must be on the top shelf, and choices B, D, E do not contain 2, you can eliminate them.

Hope that helps. Let us know if you need further explanation.

Ricky Hutchens

Get the most out of your LSAT Prep Plus subscription.

Analyze and track your performance with our Testing and Analytics Package.